For the function f(x)=[tex]x^{2} +3x-18[/tex] solve the following. f(x)[tex]\geq 0[/tex]

Answers

Answer 1
x= 36, solved using the quadratic formula
-b + sqrt b^2 -4ac / 2a
-3 + 3 -4 (-18) / 2
72 / 2
36

Related Questions

A sampling distribution of the sample means of many large samples of a probability distribution is normally distributed. Which statement is true according to the Central Limit Theorem?
A the original PD can have any shape
B the original PD must also be normal
C the original PD must be uniform
D the original PD must be symmetric

Answers

The statement that is true according to the Central Limit Theorem is B. the original PD must also be normal.

Which is the Central Limit Theorem?

According to the central limit theorem, if you take sufficiently large samples from a population, the means of the samples will be normally distributed even if the population is not normally distributed. The central limit theorem (CLT) states that as sample size increases, the distribution of sample means approaches a normal distribution, regardless of the distribution of the population.

The central limit theorem says that the sampling distribution of the mean will always be normally distributed, as long as the sample size is large enough.

In conclusion, the correct option is B.

Learn more about central limit on:

https://brainly.com/question/18403552

#SPJ1

The map of a walking trail is drawn on a coordinate grid with three points of interest. The trail starts at R(−3, 2) and goes to S(2, 2) and continues to T(2, −5). The total length of the walking trail is
units. (Input whole numbers only.) (3 points)

Answers

Answer:

12

Step-by-step explanation:

a box contains ten sealed envelopes numbered 1, . . . , 10. the first six contain no money, the next two each contains $5, and there is a $10 bill in each of the last two. a sample of size 3 is selected with replacement (so we have a random sample), and you get the largest amount in any of the envelopes selected. if x1, x2, and x3 denote the amounts in the selected envelopes, the statistic of interest is m

Answers

The probability distribution for the statistic is ; for m = 0 , P(0) =  0.216  , for m = 5 ,P(5) = 0.296 and for m = 10 , P(10) = 0.488  .

In the question ,

it is given that ,

the box has 10 sealed envelopes , that contains number from 1,2,...,10 .

the first six envelopes has no money .

next two envelopes has $5

and the last two contains $10 bill .

if x₁, x₂, and x₃ denote the amounts in the selected envelopes,

the statistic of interest is m = maximum of X₁ , X₂ and X₃ .

we need to find probability distribution for the statistic m,

So , P(m = 0) = P( all three envelope contains no money) = (6/10)³ = 0.216

and P(m = 5) = P( all from 8 envelope which contains at most $5) - P(all three envelope with 0 money)

= (8/10)³ - 0.216

= 0.296

and P(m = 10) = 1 - (8/10)³ = 0.488

Therefore , the required probability is P(0) = 0.216 ,   P(5) =  0.296 and P(10) = 0.488  .

The given question is incomplete , the complete question is

A box contains ten sealed envelopes numbered 1, . . . , 10. the first six contain no money, the next two each contains $5, and there is a $10 bill in each of the last two. a sample of size 3 is selected with replacement (so we have a random sample), and you get the largest amount in any of the envelopes selected. if x₁, x₂, and x₃ denote the amounts in the selected envelopes, the statistic of interest is M = the maximum of X₁ ; X₂ , and X₃ .  Obtain the probability distribution of this statistic ?

Learn more about Probability here

https://brainly.com/question/11938528

#SPJ4

In the Two Period Consumer model, the Net Effect from a decrease in the interest rate for a net borrower is:
Group of answer choices
C1↓; C2?; Savings ↑
C1↑; C2?; Savings ↓
C1↑; C2↓; Savings ↓
C1?; C2↓; Savings ?
C1?; C2↑; Savings ?

Answers

In the two period consumer model , the net effect from a decrease in the interest rate for a net borrower is increase in present consumption (C1 increases) , future consumption decreases (C2 decreases) and savings decreases.

What is two period consumer model?

The consumer lives for two cycles before passing away. Therefore, there is no use in saving during the second time period. Rearranging, our present disposable income minus our current consumption is what we call savings.By deciding not to spend their entire income during the present period, consumers conserve money. They borrow money from the credit market to cover the difference between their income and current expenditure, boosting their future period income by the amount they saved plus interest.A useful simplification, in fact. Maximum current expenditure exceeds maximum current income.

Hence, the net effect from a decrease in the interest rate for a net borrower is increase in present consumption (C1 increases) , future consumption decreases (C2 decreases) and savings decreases.

To know more about income check the below link:

https://brainly.com/question/28735352

#SPJ4

the sugar sweet company delivers sugar to its customers. let be the total cost to transport the sugar (in dollars). let be the amount of sugar transported (in tons). the company can transport up to tons of sugar. suppose that gives as a function of . identify the correct description of the values in both the domain and range of the function. then, for each, choose the most appropriate set of values.

Answers

The domain of the function is: [0,∞), the range of the function is: [3500,∞)

What does domain and range mean?

Domain and Range: The set which contains all the first elements of all the ordered pairs of relation R is known as the domain of the relation. The set which contains all the second elements, on the other hand, is known as the range of the relation.

The potential input and output values of a function are its domain and range, respectively.

The formula for the function is C=130S+3500.

the region

This is a list of the function's potential S values.

Due to the fact that S refers to a physical quantity, it cannot be less than 0. (i.e. tons of sugar)

The domain of the function is [0,∞)

Since S can have any value larger than 0.

The variety

This represents the function's potential C values.

The formula for the function is C=130S+3500.

Assuming S = 0, the following is true: C=130*0+3500=3500.

C must be more than 3500.

Any number higher than 3500 can be used as the value of C.

Consequently, the function's range is [3500,∞)

To learn more about Domain and Range visit:

brainly.com/question/28135761

#SPJ4

test for an association between handeness and career for these 5 professions. state the null and alternative hypothesis

Answers

Null Hypothesis: There is no association between handedness and career for Accountants. And Alternative Hypothesis: There is an association between handedness and career for Accountants.

Profession 1: Lawyer

Null Hypothesis: There is no association between handedness and career for Lawyers.

Alternative Hypothesis: There is an association between handedness and career for Lawyers.

Profession 2: Pilot

Null Hypothesis: There is no association between handedness and career for Pilots.

Alternative Hypothesis: There is an association between handedness and career for Pilots.

Profession 3: Accountant

Null Hypothesis: There is no association between handedness and career for Accountants.

Alternative Hypothesis: There is an association between handedness and career for Accountants.

Profession 4: Nurse

Null Hypothesis: There is no association between handedness and career for Nurses.

Alternative Hypothesis: There is an association between handedness and career for Nurses.

Profession 5: Teacher

Null Hypothesis: There is no association between handedness and career for Teachers.

Alternative Hypothesis: There is an association between handedness and career for Teachers.

Learn more about null hypothesis here

https://brainly.com/question/28920252

#SPJ4

Which is the equation of line b

Answers

The equation of line b that passes through the point (-3,2) is  y=-1/4x+5/4

How do you find a line equation?

to get the equation of a line given a point and its slope.

Recognize the slope.

Decide on the point.

In the point-slope form, y y 1 = m (x x 1), substitute the values. y − y 1 = m (x − x 1) .

Slope-intercept form should be used to write the equation.

What are the 3 different equations of lines?

There are three major forms of linear equations: point-slope form, standard form, and slope-intercept form.

In the above question we see that,

The line b passes through the point (-3,2)

the equation of line should satisfy this point

In the given option the only line that satisfy the point is

option C

y=-1/4x+5/4

hence option C ,y=-1/4x+5/4 is the answer

To learn more about lines, click the following link:-

https://brainly.com/question/18831322

#SPJ1

Graph the system of inequalities.
y>4 x-3
3 y-x≤9
Use the graphing tool on the right to graph the system of inequalities.

Answers

The graph of the inequality y > 4x - 3  , 3y - x ≤ 9 . is shown below.

In the question ,
the inequalities are given ,

we have to graph given inequalities ,

the inequalities are y > 4x - 3

3y - x ≤ 9 .

in the first inequality , y > 4x - 3

we put x = 0 , y is = -3 .

we put y = 0 , x is = 3/4 .

So ,the points for the line will be  (0,-3) and (3/4,0) .

since the inequality does not have equal to  sign , the line will be a dotted line .

in the second inequality ,3y - x ≤ 9 .

we put x = 0 ,  y is = 3 .

we put y = 0 ,  x is = -9 .

the points for the line will be (0,3) and (-9,0).

Since the inequality has equal to sign . Thus, line will be solid line .

Therefore , the for the given inequality is shown below.

Learn more about Inequality here

https://brainly.com/question/22922637

#SPJ4

As part of her science project , Shay is making a model of a wind farm.She wants to put 24 turbines in her model.What arrays can Shay make using 24 turbines

Answers

Answer: In Shay's wind farm their can be 4 arrays

Search up the meaning of a Array if you are confused

Step-by-step explanation:

24 has 4 pairs of factors.

1,242,123,84,6

Ben and Bob made a snowman been spent two hours more than double the time Bob spent

Answers

If you need to make an equation, this is how I would do it.
x = the time it took Bob to make a snowman
y = the time it took Ben to make a snowman
(but the variables can be any letters)

Because Ben spent 2 hours more than 2 times the time Bob spent, the equation would be:
y = 2x + 2

Hope this helps

A piece of cardboard is 5 centimeters longer than it is tall. The area of the piece of cardboard is 66 square centimeters. How many centimeters long is the piece of cardboard?

Answers

Answer:

12.244 cm long

Step-by-step explanation:

The area of a rectangle is equal to its height times its width, so if the piece of cardboard has an area of 66 square centimeters and is 5 centimeters longer than it is tall, we can write the following equation to represent the dimensions of the piece of cardboard:

height * (height + 5) = 66

To solve this equation, we can first divide both sides of the equation by the height of the piece of cardboard to get:

height + 5 = 66 / height

We can then subtract 5 from both sides of the equation to get:

height = 66 / height - 5

We can then multiply both sides of the equation by the height to get:

height^2 = 66 - 5 * height

We can then use the quadratic formula to solve for the height of the piece of cardboard:

height = (-5 +/- sqrt(5^2 - 4 * 1 * -66)) / (2 * 1)

The value of the square root in the quadratic formula must be positive, so we take the positive value for the height:

height = (-5 + sqrt(5^2 + 4 * 1 * 66)) / (2 * 1)

This simplifies to:

height = (5 + sqrt(61)) / 2

The height of the piece of cardboard is approximately 7.244 centimeters. Since the piece of cardboard is 5 centimeters longer than it is tall, it must be 5 + 7.244 = 12.244 centimeters long.

Therefore, the piece of cardboard is approximately 12.244 centimeters long.

Find the cosecant. Someone help me asap pls!

Answers

Answer:

sqrt2

Step-by-step explanation:

csc (x)= 1/sin(x)

sin(x)= (sqrt22)/(sqrt22*sqrt2)= 1/sqrt2

1/(1/sqrt2)=sqrt2/1=sqrt2

No matter what I've tried, nothing works. Hopefully you can help me

Answers

Answer:

see explanation

Step-by-step explanation:

given Δ ABD is isosceles with AB = AD then base angles are congruent, so

∠ ADB = ∠ ABD = 72°

then

∠ BAD = 180° - (72 + 72)° = 180° - 144° = 36° ( sum of angles in a triangle )

given Δ ABC is isosceles , then

∠ BCD = ∠ BAD = 36°

the exterior angle of a triangle is equal to the sum of the 2 opposite interior angles.

∠ ADB is an exterior angle of Δ BCD , then

∠ BCD + ∠ DBC = ∠ ADB

36° + ∠ DBC = 72° ( subtract 36° from both sides )

∠ DBC = 36°

So ∠ BCD = ∠ DBC = 36°

Thus the base angles of Δ BCD are congruent , then

Δ BCD is isosceles with BD = CD

Which of the following is true about the t-distribution? Assume a reference (random) sample of size 100 with sample mean 10 and sample standard deviation of 1 with unknown population mean and variance.
a. The t-distribution will look similar to the standard normal distribution as the sample size increases which results in the degrees of freedom increasing.
b. The t-distribution will look similar to the standard normal distribution as the sample size increases which results in the degrees of freedom decreasing.
c. The t-distribution will look similar to the normal distribution with mean 10 and standard deviation 1 as the sample size increases which results in the degrees of freedom decreasing.
d. The t-distribution will look similar to the normal distribution with mean 10 and standard deviation 1 as the sample size increases which results in the degrees of freedom increasing.
e. None of these

Answers

1000 times were added to the n=30 sample. Even with n=30, the sample variance distribution is still very biased.

What is probability ?

Probability refers to possibility.

A random event's occurrence is the subject of this area of mathematics. The range of the value is 0 to 1. Mathematics has incorporated probability to forecast the likelihood of various events.

The degree to which something is likely to happen is basically what probability means.

You will understand the potential outcomes for a random experiment using this fundamental theory of probability, which is also applied to the probability distribution.

Knowing the total number of outcomes is necessary before we can calculate the likelihood that a specific event will occur.

According to our question-

The sample variance's variance (SD) is still very large. The range is from 20 to 30%.

Because we are utilizing estimated population variance rather than the genuine population variance, which has some degree of uncertainty, the e t distribution has a longer tail when degrees of freedom are limited. Given that the estimated population variance is nearer to the actual population variance when sample size = 30, the t distribution will be closer to the z distribution.

learn more about probability click here:

brainly.com/question/13604758

#SPJ4

Compute Shannon's %NMC. Round your answer to the nearest percent. For example, if your
computed answer is 99.4% your answer rounds down to 99%. If your computed answer is
99.5% you should round up to 100%.

Answers

The Shannon's %Net market contribution = 40%

What is NMC (Net market contribution)?

Net market contribution (NMC) is equal to total sales revenue less total costs (except market costs) minus market costs.

Cost overall (not including market expenses):

150000 packaged (in cans and bottles).

Kegs =80000

WIP Loss/Reduction = 42000

Labor Contracts = 9000

Total Direct Labor: 240000

18000 inbound pounds

180000 in compensation

Total= 719000

Consumer advertising costs 26850 on the market.

Marketing for trade = 31761

Promotional Sales = 18000

Total=76611 Net market contribution=1320271-719000-76611=$ 524660 %NMC=524660/1320271=0.3974=39.74%=40%

To learn more about the Net market contribution from the given link

https://brainly.com/question/16829947

#SPJ1

Please answer the two following questions.

Answers

The transformed graph is attached

The domain is (-∝, ∝) and the range is (-3, ∝)

How to transform the graph?

From the question, we have the following parameters that can be used in our computation:

y = eˣ

To transform the graph to y = e⁻ˣ - 3, we make use of the following rules:

Reflect across the y-axisShift down by 3 units

See attachment for the graph of y = e⁻ˣ - 3

The domain and the range

From the attached graph, we have:

The graph extends across the x-axis i.e. domain = (-∝, ∝)

The graph opens upward from y > -3 i.e. range = (-3, ∝)

Read more about domain and range at

https://brainly.com/question/2264373

#SPJ1

A stage manager is trying to seat important guests in the front row of a theater. She would like to seat a diplomat in the first seat, a singer in the second seat, and a movie director in the third seat. If there are 3 diplomats, 2 singers, anf 2 directors attending the show, how many different front row plans are possible? (The workbook got the answer of 288) How?

Answers

Answer:

12 row plans

Step-by-step explanation:

the first seat has two possible options , the two directors. The second seat has another two options. With each director comes 2 options for the second seat so 2×2=4 possible options for the first and second seat . The 3rd seat has three options, that means for each combination of the 1st and 2nd seat three possible options 2×3=12 combinations

What is 1 1/2 + 5/7?

Answers

3/2+5/7
31/14

D.) 2 3/14

Mark me Brainly pls
Answer:2 3/14


Explanation:
You would start by turning the improper fraction being 1 1/2 into 3/2

Then you would need for both denominators to be the same number and in order for that to be the case you need to find the Least Common multiple which is 14.

3*7/2*7 + 5*2/7*2
It’ll then turn out to be:

21/14+10/14

Since they now have the same denominator you can add them and give you the answer:

31/14

But if you want it to be improper fraction then it’ll be:
2 3/14

What properties of equality do you need to use to solve the equation 2.1x−1.6=6.8​? Solve the equation. Question content area bottom Part 1 What properties of equality do you need to use to solve the​ equation? Select all that apply. A. The Division Property of Equality B. The Multiplication Property of Equality C. The Subtraction Property of Equality D. The Addition Property of Equality Part 2 The solution is enter your response here. ​(Type the value of​ x.)

Answers

it is 6.1

Step-by-step explanation:



Overtime Hours Worked A random sample of 15 registered nurses in a large hospital showed that they worked on average 44.6
hours per week. The standard deviation of the sample was 2.3. Estimate the mean of the population with 99% confidence.
Assume the variable is normally distributed. Round intermediate answers to at least three decimal places. Round your final
answers to one decimal place.

Answers

Answer:

With 99% confidence we can say that the mean of the population is between 43.1 and 46.1

Step-by-step explanation:

e. 2|x-4|-3y³ when x=3 and y=-2

Answers

Answer:

26

Step-by-step explanation:

To find the value of the expression 2|x-4|-3y³ when x=3 and y=-2, we need to substitute the values of x and y into the expression and then simplify. The absolute value part of the expression, |x-4|, is the distance between the number x and 4 on the number line. When x=3, this distance is |3-4|=1. So the expression becomes 2*1-3(-2)³ = 2-3(-8) = 2+24 = 26. Therefore, when x=3 and y=-2, the value of the expression 2|x-4|-3y³ is 26.

You can use induction. Here's a very very rusty sketch of the proof.
Given -
Union of two regular languages is regular.
Let f(n) be a function representing the union of n regular languages.
Question Is f(n) a regular language?

Answers

Lk+1 is the k+1th regular language, for example. The presented hypothesis states that f(n) = f(k+1) is regular since f(k) and Lk+1 are both regular.

What is the purpose of the inductive hypothesis?

The induction hypothesis's function The assumption made at the beginning of the induction phase is known as the induction hypothesis, and it is the situation where n = k for the proposition we are trying to prove ("P(k)"). If you don't bring this hypothesis into play at any point throughout the proof of the induction phase, something is amiss.

Given:

It is normal for two regular languages to be combined.

Give the function f(n) the name "union of n regular languages."

Question Is the language f(n) regular?

The union of a single regular language is regular

if n = 1. Base Case

The provided hypothesis tells us that f(2) is regular

if n = 2.

Assume that f(n) is regular for every n = k.

This is the inductive hypothesis.

Let n equal k+1 in the inductive step. The inductive premise tells us that f(k) is a regular language.

Consequently, f(n) = f(k+1) = Lk+1 U f(k) (k)

Lk+1 is the k+1th regular language,

for example. The presented hypothesis states that f(n) = f(k+1) is regular since f(k) and Lk+1 are both regular.

To know more about inductive hypothesis visit:

https://brainly.com/question/29525504

#SPJ4

can anyone please help me with this math question?

Answers

Answer:

a) (7-2x) +3 = 10 - 2X

b) x = y + 3 <=> y = x-3

c) (x-3).-2 +7 = -2x +6 +7 => -10 + 13 = 3

Find the volume V of the solid obtained by rotating the region bounded by the given curves about the specified line. y = 27x^3, y = 0, x = 1; about x = 2

Answers

The volume V of the solid is obtained by rotating the region bounded by the given curves about the specified line. y = 27x^3, y = 0, x = 1; about x = 2 is 81π/5

Shell method:

The shell method is an integration for calculating the volume of a solid of revolution when integrating along an axis perpendicular to the axis of revolution. This is in contrast to disc integration which integrates along the axis parallel to the axis of revolution.

V=2π∫01(2-x). 27x³dx

  = 2π.27([tex]x^{4}[/tex]/2 - [tex]x^{5}[/tex]/5)01

  =54π(1/2-1/5)

  = 81π/5

Therefore The volume V of the solid is obtained by rotating the region bounded by the given curves about the specified line. y = 27x^3, y = 0, x = 1; about x = 2 is 81π/5

Learn more about integration:

https://brainly.com/question/27419605

#SPJ4

-1/2h(2h+3)=0 answer

Answers

Step-by-step explanation:

-1/2h(2h+3) = 0

Seperate the equation:

-1/2 = 0

h = 0

(2h+3) = 0

Subtract 3 from both sides:

2h+3 = 0

2h+3-3 = 0

Simply:

2h = -3

Final Answer:

h = 0

h = -3/2

Determine how many times the innermost loop will be iterated when the algorithm segment is implemented and run. (Assume that m and n are positive integers.)
for j := 1 to m
for k := 1 to n
[Statements in body of inner loop.
None contain branching statements that
lead outside the loop.]
next k
next j
Discrete mathematics

Answers

The inner loop iterates 'n' times, outer loop iterates 'm' times and total loop iterates (m×n) times.

Given program,

for j = 1 to m

   k = 1 to n

statement in the body of innerloop. None contain branching statements that lead outside the loop.

next j

next k

Here, in the inner loop k iterates from 1 to n that is n times and j iterates from 1 to m that is m times.

Finally, the whole loop iterates m×n times because for every value of j from 1 to m, k value increases from 1 to n.

So, the inner loop iterates 'n' times, outer loop iterates 'm' times and total loop iterates (m×n) times.

To know more about Iterations refer to:

http://brainly.com/question/28134937

#SPJ4

Escriba la ecuación en el formato y = mx + b, dada la siguiente información: Pasa por el punto A(7,2) es paralela a

3x-y=8

Answers

Answer:

El ecuación es y = 3x + 2

Step-by-step explanation:

Lo primero que tienes que hacer es poner 3x - y = 8 en la forma y = mx + b para para saber que 'm'. Porque la ecuación que intentamos escribir es paralela a 3x - y = 8, entonces tienen la misma pendiente (m).

3x - y = 8

-3x        -3x

-y = -3x + 8

/-1    /-1     /-1                   dividir todo por -1 para hacer 'y' positivo.

y = 3x - 8

Y como sabemos que la ecuación pasa por (7, 2), sabemos que pasa por 2 en el y-axis (vertical). Entonces 2 es el intercepto en y (b).

Solve for x.
3/7x +1/4x =38

Answers

Answer:

x = 56

Step-by-step explanation:

[tex]\frac{3}{7}[/tex] x + [tex]\frac{1}{4}[/tex] x = 38  The common denominator would be 28

[tex]\frac{12}{28}[/tex] x + [tex]\frac{7}{28}[/tex] x =38                                                                                                                                                

[tex]\frac{19}{28}[/tex] x = 38  Multiply both sides by [tex]\frac{28}{19}[/tex]

[tex](\frac{28}{19})[/tex][tex]\frac{19}{28}[/tex] x = [tex](\frac{28}{19})[/tex][tex]\frac{38}{1}[/tex]  Cross cancel the 19 and 38.  

x = [tex](\frac{28}{1})[/tex][tex]\frac{2}{1}[/tex]

x = 56

x=56

Explanation: simplify then isolate the variable

Count the best-case number of + operations performed by the following pseudocode segment. Assume that all possible data sets are equally likely. Preconditions: X = {x1, x2, x3, x4, x5} ⊆ {10, 20, 30, 40, 50, 60, 70, 80}, where x1 < x2 < x3 < x4 < x5. t ← 0 i ← 1 while t < 101 do t ← t + xi i ← i + 1

Answers

For the best cases there will be 6+operations, The number of operations are best cases 6 and the worst cases are 10.

Given that,

The following pseudocode snippet performs the maximum number of + operations. Assume that the probability of each potential piece of data is equal. Preconditions: X = {x₁, x₂, x₃, x₄, x₅} ⊆ {10, 20, 30, 40, 50, 60, 70, 80}, where x1 < x2 < x3 < x4 < x5. t ← 0 i ← 1 while t < 101 do t ← t + xi i ← i + 1

We know that,

Here,

X = {x₁, x₂, x₃, x₄, x₅} ⊆ {10, 20, 30, 40, 50, 60, 70, 80}

By doing the iteration method

Iteration process till 4th iteration we get 6

Therefore, For the best cases there will be 6+operations, The number of operations are best cases 6 and the worst cases are 10.

To learn more about number visit: https://brainly.com/question/17429689

#SPJ4

PLEASE HELP ME ASAP!!!
Consider the following quadratic function.

Answers

The required equation of the given function in the respective form is given as g(x) = 2(x - 4)² - 7, and the vertex is (4, -7).

What is the graph?

The graph is a demonstration of curves that gives the relationship between the x and y-axis.

Here,
g(x) = 2x² - 16x + 25
g(x) = 2[x² - 8x] + 25
g(x) = 2[x² - 8x + 16 -16] + 25
g(x) = 2(x - 4)²  - 32 + 25
g(x) = 2(x - 4)² - 7

Now, the vertex is given as (h, k) = (4, -7).
And the graph of the given function is shown.

Thus, the required equation of the given function in the respective form is given as g(x) = 2(x - 4)² - 7, and the vertex is (4, -7).

Learn more about graphs here:

brainly.com/question/16608196

#SPJ1

Other Questions
a flatter sras curve would make the phillips curve , indicating a short-run increase in inflation following an expansionary monetary and/or fiscal policy. t/f The agreement between the United States, Canada, and Mexico that merges these threecountries into one marketplace is calleda. EU.b. MERCOSUR.c. APEC.d. NAFTA.e. GATT.Answer : d I imagine that, had I been granted the opportunity to deign a booktore at age twelve, the end reult would have been pretty cloe to Cliff-hanger' Book. I'm middle-aged now, but being in thi booktore made me feel like I wa bonding with my bookworm-y, free-thinking, creative twelve-year-old pirit. Cliff-hanger' i a collection of contradiction: chaotic yet ordered; vat yet cozy; full of mut and dut and that ued-book mell, yet omehow clean a a whitle. The bet part, aide from the incredible array of book, i the hidden reading nook. Mot booktore are tingy with thee, but not Cliff-hanger'. I found a particularly comfy corner, complete with a fuchia loveeat and an afghan, where I pent the better part of an hour exploring a book about the hitory of the artificial language Volapk. Which entence bet ummarize the author' poition? Mallory, a compulsive gambler, regularly attends therapeutic meetings in which there is a group facilitator and an open format. Although there are no formal program goals, she says she benefits greatly from the sense of community offered by people who share common situations in life. Nancy is attending _____.a support group Divers in Acapulco dive from a cliff that is 54 m high. If the rocks below the cliff extend outward for 16 m, what is the minimum horizontal velocity a diver must have to clear the rocks? Please select the word from the list that best fits the definition the belief that one's own race or ethnic group is superior to other races or ethnic groups. A firm could continue to operate for years without ever earning a profit as long as it is producing an output whereMR < ATC.ATC > AVC.MR > AVC.AFC < AVC. Which of the following songs would be the most challenging to arrange because of its wide melodic range and complex harmonies?A. Auld Lang SyneB. Down by the RiversideC. Danny BoyD. Amazing Grace im on ixl rn what is mean median mode and range if the total cost per day remains uncahnged in the long run what is the long run equilibrium price for umbrellas Inadequate intake of calcium and vitamin D, reduced calcium and vitamin D absorption, and the reduced synthesis of vitamin D in the skin are all factors that contribute to the development of the bone disease the war crimes trials in nuremberg and tokyo following world war ii established the concept that __________ is measured by the projects impact on uncertainty regarding the firms future returns. what do you think should be done to repair the harm done to Native Americans? electronic watches keep accurate time using crystal oscillators. inside the watch, there is a tiny block of quartz which vibrates. two opposite faces of the block move alternately toward each other and away from each other. this is a caused by a standing wave in the block. the two opposite faces are at antinodes, and the plane halfway between these two faces is at a node. if the two faces are 7.79 mm apart and the speed of sound in quartz is 3.72 km/s, find the frequency of the vibration. a series could be an arithmetic progression or geometric progression or fibonacci series.you will be provided with N numbers. you task is to identify which series it is, and find the next number in that series. Reactive Industries has the following capital structure. Its corporate tax rate is 35%.SecurityMarket ValueRequired Rate of ReturnDebt$20 Million6%Preferred stock$30 Million8%Common stock$50 Million12%What is its WACC? Martin Seligman conducted studies in which dogs were placed on a floor that administered electric shocks with no way of escaping. When the dogs were eventually given the opportunity to escape,a. most dogs remained where they were.b. most dogs jumped out right away.c. most dogs helped other dogs escape before escaping themselves.d. most dogs escaped quickly, but returned within several minutes. in accordance with the constant-growth ddm, calculate the intrinsic value of stock a based on the following information: the expected growth rate of dividends is 6% for stock a and 5% for stock b. the expectation is that each of the stocks will pay a dividend of $4 in the upcoming year. you want to earn a return of 10% on each of two stocks. explain adolescent egocentrism and how it is the same and how it is different from egocentrism seen in early childhood.